Questions tagged [analytic-number-theory]

A beautiful blending of real/complex analysis with number theory. The study involves distribution of prime numbers and other problems and helps giving asymptotic estimates to these.

Filter by
Sorted by
Tagged with
18 votes
2 answers
1k views

Can the Dedekind zeta function distinguish between real and imaginary quadratic number fields?

Suppose I am given a machine that gives me the coefficients $a_1$, $a_2$, $a_3$, ... of a Dirichlet series $$\sum_1^{\infty} \frac{a_n}{n^s} $$ and assume that I know that this Dirichlet series is the ...
Andreas Holmstrom's user avatar
18 votes
2 answers
735 views

Does the mean ratio of the perimeter to the hypotenuse of right triangles converge to $1 + \dfrac{4}{\pi}$?

Conjecture: Let $\mu_x$ be the arithmetic mean of the ratio of the perimeter to the hypotenuse of all primitive Pythagorean triplets in which no side exceeds $x$; then, $$ \lim_{x \to \...
Nilotpal Kanti Sinha's user avatar
18 votes
2 answers
1k views

Why isn't meromorphic continuation enough for converse theorems?

This is a very naive question which really does little more than highlight my ignorance of how converse theorems really work. Take an algebraic gadget which should be conjecturally associated to an ...
Kevin Buzzard's user avatar
18 votes
1 answer
1k views

The conjecture of Montgomery and Soundararajan on primes in short intervals: Empirical inconsistencies?

Assume that $y/ \log x \rightarrow \infty$ and that $y/x \rightarrow 0$. Then, from a conjecture by Montgomery and Soundararajan, we expect the number of primes in the interval $[x,x+y]$ to be ...
user45947's user avatar
  • 955
18 votes
1 answer
2k views

A question about Speiser's 1934 result on the Riemann hypothesis

A number of sources concerning Speiser's 1934 result state that the Riemann Hypothesis (RH) implies $\zeta'(s)\neq 0$ for all $0<\text{Re}(s)<1/2$. But I have seen some (possibly less reliable) ...
pbs's user avatar
  • 243
18 votes
2 answers
1k views

Lower bounds on the easier Waring problem

The easier Waring problem asks for the least number $v=v(k)$ such that every every integer is a sum of $v$ $k$'th powers with signs, i.e. every $n\in \mathbb{N}$ is of the form $$n=x_1^k\pm x_2^k\pm\...
Boris Bukh's user avatar
  • 7,746
18 votes
2 answers
2k views

Primes of the form a^2+1

The fact that the Riemann zeta function $\zeta(s)$ and its brethren have a pole at $s=1$ is responsible for the infinitude of large classes of primes (all primes, primes in arithmetic progression; ...
Franz Lemmermeyer's user avatar
18 votes
2 answers
1k views

Median largest-prime-factor

Let $P(n)$ denote the largest prime factor of $n$. For any integer $x\ge2$, define the median $$ M(x) = \text{the median of the set }\{P(2), P(3), \dots, P(x) \}. $$ Classical results of Dickman and ...
Greg Martin's user avatar
  • 12.7k
18 votes
2 answers
5k views

How did Riemann calculate the first few non-trivial zeros of the zeta-function?

Does anyone know how Riemann calculated the first few non-trivial zeros of the Zeta function? I am wondering if he approximated the integral, $\frac{1}{2 \pi i} \int_{R} \frac{{\xi}^\prime(z)}{\xi (z)...
Mustafa Said's user avatar
  • 3,679
18 votes
3 answers
2k views

A question on the prime divisors of p-1

For each positive integer n we may define the convergent sum $$ s(n)=\sum_{p}\frac{(n,p-1)}{p^2} $$ where the summation is over primes p and $(a,b)$ denotes the greatest common divisor of a,b. It is ...
Dr. Pi's user avatar
  • 2,949
18 votes
1 answer
4k views

Tightening Zhang's bound [closed]

Inspired by a blogpost by Scott Morrison and ongoing discussion there I decided to create this community wiki to track progress on the original bound of Yitan Zhang. The original bound was $70,000,...
18 votes
1 answer
1k views

Distinct simple zeros of Dirichlet L-functions

Given a finite set of distinct primitive Dirichlet characters, $\chi_1, \dots, \chi_r$, is it known that the product of the L-functions, $$L(s):=\prod_{i=1}^r L(s,\chi_i),$$ has a simple zero? It's ...
rlo's user avatar
  • 1,671
18 votes
2 answers
1k views

The Riemann zeta function and Haar measure on the profinite integers

In an answer to a question on MU about the Riemann zeta function, I sketched a proof that the probability distribution on $\mathbb{N}$ which assigns $n$ the probability $$\frac{ \frac{1}{n^s} }{\zeta(...
Qiaochu Yuan's user avatar
18 votes
2 answers
3k views

Zeta-function regularization of determinants and traces

The short answer to my question may be a pointer to the right text. I will give all the background I know, and then ask my questions in list form. Let A be an operator (on an infinite-dimensional ...
Theo Johnson-Freyd's user avatar
18 votes
1 answer
1k views

Does summing divergent series using cutoff functions give consistent results?

One way to try to give a value $S$ to a divergent series $\sum_{n=1}^\infty a_n$ is with a smooth cutoff function: $$ S = \lim_{N\to\infty}\sum_{n=1}^\infty a_n \eta\left(\frac{n}{N}\right) $$ where $\...
not all wrong's user avatar
18 votes
1 answer
2k views

Where might I find a scanned handwritten copy of Ramanujan's second letter to Hardy?

I am giving a lecture to undergraduates on the lovely identity $$1 + 2 + 3 + 4 + \cdots = -\frac{1}{12}.$$ Ramanujan wrote in his second letter to Hardy (courtesy Wikipedia), "Dear Sir, I am very ...
Frank Thorne's user avatar
  • 7,199
18 votes
0 answers
598 views

Consecutive integers of the form $2^a 3^b 5^c$

Let $\mathcal{N}$ denote the set of all products of (powers of) $2,3$ and $5$: $$ \mathcal{N} = \{ 2^a 3^b 5^c \ : \ a,b,c \geq 0 \} \subset \mathbb{N}.$$ We use the elements of $\mathcal{N}$ to ...
Jakub Konieczny's user avatar
18 votes
0 answers
711 views

Infinite extensions such that every elliptic curve has finite rank

The comments to this answer seem to make the following claim. Claim. Let $K$ be the maximal abelian extension of $\mathbf Q$ that is unramified away from $p$ (more generally, away from a finite set $S$...
R. van Dobben de Bruyn's user avatar
17 votes
3 answers
3k views

A variant of the Goldbach Conjecture

I am asking if this variant of the weak Goldbach Conjecture is already known. Let $N$ be an odd number. Does there exist prime numbers $p_1$, $p_2$ and $p_3$ such that $p_1+p_2-p_3=N$? Ideally, can ...
Omid Hatami's user avatar
17 votes
2 answers
2k views

Is this equivalent to RH - Riemann hypothesis?

$$\pi = 3\prod_{\zeta(1/2+it) = 0}\frac{9+4t^2}{1+4t^2}\iff\text{RH is true}.$$
Dimitris Valianatos's user avatar
17 votes
4 answers
3k views

Using Quotient of Prime Numbers to Approximation Reals

We know a positive rational number can be uniquely written as $m/n$ where $m$ and $n$ are coprime positive integers. Particularly, we can pick out those numbers with $m$ and $n$ both prime. Question ...
Ash GX's user avatar
  • 273
17 votes
3 answers
2k views

About the prime divisors of values of polynomials

Let $P$ be a polynomial having integer coefficients (and degree $\geq 3$), and let $\mathscr P_P$ be the set of prime numbers dividing some value $P(n)$ with $n \in \mathbb Z$. Is it true that $\...
Konstantinos Gaitanas's user avatar
17 votes
2 answers
3k views

Consequences of the Birch and Swinnerton-Dyer Conjecture?

Before asking my short question I had made some research. Unfortunately I did not find a good reference with some examples. My question is the following What are the consequences of the Birch and ...
17 votes
3 answers
1k views

PNT for general zeta functions, Applications of.

When I read it for the first time, I found the whole slog towards proving the Prime Number Theorem and the final success to be magnificent. So I am curious about more general results. We talk of ...
Anweshi's user avatar
  • 7,272
17 votes
1 answer
978 views

Theta functions, re-expressed

Recall the classical $\theta(q):=\prod_{k=1}^{\infty}(1-q^k)$ and define the sequences $a_n$ and $b_n$ by $$\frac{\theta^3(q)}{\theta(q^3)}=\sum_{n=0}^{\infty}a_nq^n \qquad \text{and} \qquad F(q):=\...
T. Amdeberhan's user avatar
17 votes
3 answers
3k views

Largest known zero of the Riemann zeta function

Numerical calculations on the zeroes of the Riemann zeta function have reached a very high degree of refinement and sophistication and I think that the first $10^{20}$ (with positive imaginary part) ...
Bazin's user avatar
  • 15.2k
17 votes
1 answer
673 views

Probability that $n$ is coprime to both $m$ and $m+1$

It is well known that the set $\{(n,m) \in \Bbb N^2 : \gcd(n,m) = 1\}$ of coprime integers has a natural density of $\zeta(2)^{-1}$ in $\Bbb N^2$. It seems reasonable to think that the density of the ...
Siméon's user avatar
  • 635
17 votes
1 answer
3k views

What is the relation between Quasicrystals, Riemann Hypothesis, and PV numbers?

Could somebody explain to me, from a mathematical stand-point, what is a quasi-crystal, and how it relates to the set of Pisot numbers, and the Riemann Hypothesis? I've heard Freeman Dyson say that ...
kolik's user avatar
  • 293
17 votes
2 answers
2k views

Is every odd positive integer of the form $P_{n+m}-P_n-P_m$?

I am looking for a comment, reference, remark, or proof of three conjectures as follows: Conjecture 1: Let $x$ be an odd positive integer. Then there exist two integers $n, m \ge 2$ so that $$x=P_{n+...
Đào Thanh Oai's user avatar
17 votes
1 answer
3k views

Values of zeta at odd positive integers and Borel's computations

Someone recently quoted to me this recent article that claims to prove that $\zeta(2n+1) \notin (2\pi )^{2n+1} \mathbb{Q}$. [Edit: published reference: Musha, Takaaki. Negation of the conjecture for ...
AFK's user avatar
  • 7,387
17 votes
2 answers
4k views

On Siegel mass formula

I have asked this question exactly here. The question is as follows: I am interested deeply in the following problem: Let $f$ be a (fixed) positive definite quadratic form; and let $n$ be an ...
Davood Khajehpour's user avatar
17 votes
1 answer
1k views

A converse of the abc conjecture?

Let ${\rm rad}(n)$ denote the radical of a positive integer $n$, i.e. the product of its distinct prime divisors. Given positive integers $a$ and $b$, the triple $(a,b,a+b)$ is called an abc triple if ...
Stefan Kohl's user avatar
  • 19.5k
17 votes
3 answers
1k views

Is there a non-constructive proof that a specific integer satisfies the Goldbach conjecture?

This is a question expecting the answer no. I'm wondering out of curiosity whether there is any positive integer $n$ for which it is known that $2n$ is a sum of two primes, but which is such that no ...
gowers's user avatar
  • 28.7k
17 votes
2 answers
1k views

Which L-functions are not "Langlands-Shahidi L-functions"?

The Langlands-Shahidi method, among other things, obtains certain L-functions from the constant term of Eisenstein series attached to so-called $(G,M)$ pairs, where $G$ is a reductive group, $M$ a ...
Tian An's user avatar
  • 3,709
17 votes
2 answers
1k views

Chen's Theorem with congruence conditions.

I would like to revisit a closed question of asterios in a more MO kind of way, because it cuts quite close to a related question about sieving that might be of general interest. The original ...
user avatar
17 votes
1 answer
576 views

Smoothed exponential sums: bounds and sources?

Let $f:\mathbb{R}\to\mathbb{C}$ be differentiable $k$ times, with $f, f',\dotsc,f^{(k)}\in L^1$. Let $\alpha\in \mathbb{R}/\mathbb{Z}$, $\alpha\ne 0$. In "Every odd number..." (Math. Comp. 83, 2014), ...
H A Helfgott's user avatar
  • 19.4k
17 votes
1 answer
1k views

On the Hasse-Weil L-function of $P^n$

So let us start with the "simplest" scheme over $Spec(\mathbf{Z})$ namely $X_0=Spec(\mathbf{Z})$. Then the (reciprocal) Weil zeta function of $X_0$ at a prime $p$ is given by $Z_p(T)=1-T$ (a ...
Hugo Chapdelaine's user avatar
16 votes
5 answers
3k views

Historical question in analytic number theory

The analytic continuation and functional equation for the Riemann zeta function were proved in Riemann's 1859 memoir "On the number of primes less than a given magnitude." What is the earliest ...
David Hansen's user avatar
16 votes
5 answers
3k views

Resources where I can find open problems in number theory along with their level of difficulty

NOTE: I will not accept an answer because a lot of answers are really good and if anyone want to post under this question later then they are most welcome to post as comment or answer because it will ...
16 votes
4 answers
2k views

Who first proved that there are at least n^(1-ε) primes up to n?

It's well-known that Hadamard and de la Vallée-Poussin independently proved the Prime Number Theorem in 1896: that $\pi(n)=n/\log n+o(n/\log n)$. I'm curious as to a weaker result: that for any $\...
Charles's user avatar
  • 8,994
16 votes
2 answers
3k views

Many representations as a sum of three squares

Let $r_3(n) = \left|\{(a,b,c)\in {\mathbb Z}^3 :\, a^2+b^2+c^2=n \}\right|$. I am looking for the maximum asymptotic size of $r_3(n)$. That is, the maximum number of representations that a number can ...
Adam Sheffer's user avatar
  • 1,052
16 votes
4 answers
2k views

What can be said about this double sum?

Question. Can this number be expressed in terms of classical values? $$\sum_{n,m=1}^{\infty}\frac1{(n^2+m^2)^{\frac32}}=1.056348517615643291\dots$$ UPDATE. I'm encouraged by Noam, Kevin and Igor's ...
T. Amdeberhan's user avatar
16 votes
4 answers
9k views

Exact formulas for the partition function?

I am curious, what kind of exact formulas exist for the partition function $p(n)$? I seem to remember an exact formula along the lines $p(n) = \sum_k f(n, k)$, where $f(n, k)$ was some extremely ...
Frank Thorne's user avatar
  • 7,199
16 votes
1 answer
992 views

Tight prime bounds

This is a cross-post of this question on MSE. I would not usually do this, but have decided to in this case since it has had no responses having been posted as a bounty question. I did not delete the ...
martin's user avatar
  • 1,893
16 votes
2 answers
2k views

Could this unexpected bias in the distribution of consecutive primes have any impact on the security of encryption algorithms?

In a recent paper a quite unexpected result about a new pattern in prime numbers emerged: Unexpected biases in the distribution of consecutive primesby Oliver, R. J. L.; Soundararajan, K. (Submitted ...
vonjd's user avatar
  • 5,875
16 votes
4 answers
2k views

Arithmetic progressions without small primes

The following question came up in the discussion at How small can a group with an n-dimensional irreducible complex representation be? : Is it known that there are infinitely many primes p for which ...
David E Speyer's user avatar
16 votes
2 answers
687 views

Comparing sizes of sets of natural numbers

It seems natural to consider $\lim_{q \rightarrow 1^-} \sum_{n \in S} q^n - \sum_{n \in T} q^n$, when it exists, as a way of comparing the sizes of two sets $S,T \subseteq {\bf N}$ that have the same ...
James Propp's user avatar
  • 19.4k
16 votes
2 answers
2k views

Ordinary Generating Function for Mobius

Is there any information known for the Ordinary Generating Function for Mobius? $$ \sum_{n=1}^{\infty} {\mu(n)}x^n $$ I know that It has radius of convergence 1. Does not have limit as $x\rightarrow ...
Sungjin Kim's user avatar
  • 3,310
16 votes
1 answer
666 views

Dirichlet series with a single zero

I need to find a Dirichlet series f that has the following property. f is zero in only one point s such that Re(s) > $\sigma_c $.
Clueless's user avatar
  • 161
16 votes
1 answer
4k views

Order of magnitude of $\sum \frac{1}{\log{p}}$

Question: What is the order of magnitude of the following sum? $$ \sum_{\substack{p<n\\\text{$p$ prime}}} \frac{1}{\log{p}} $$ Additional information: Since $$ \sum_{\substack{p<n\\\text{...
Daniel Soltész's user avatar

1 2 3
4
5
59